¿Por qué un escalar real no puede acoplarse al campo electromagnético?

Si tenemos un escalar complejo ϕ sabemos que la interacción de calibre invariante con A es dado por A m j m , dónde j es la corriente de Noether del tu ( 1 ) simetría del lagrangiano

(1) j m ϕ m ϕ ϕ m ϕ

Si tenemos un escalar real en su lugar, j = 0 y el campo no acopla A : las partículas reales no están cargadas.

Mi pregunta es: ¿y si tomamos

(2) j m ϕ m ϕ
poner en pares ϕ a A ? La interacción es invariante de Lorentz y renormalizable, pero no es invariante de calibre, lo que probablemente sea algo malo. ¿En qué punto se rompería esta teoría?

Probablemente esto sea muy ingenuo, pero creo que las reglas de Feynman para esta teoría son sencillas. Así que supongo que la teoría tiene sentido al menos perturbativamente. La teoría probablemente tiene fallas en un nivel más fundamental, pero parece que no puedo encontrar dónde (¿algún tipo de anomalía de calibre, tal vez?)

El hecho de que la teoría no sea invariante de calibre implica que todos los grados de libertad de A m debe tener un significado físico: esta no es la teoría de los fotones donde solo los grados de libertad transversales tienen sentido.
@ValterMoretti y por lo tanto el componente de tiempo A 0 sería un campo real y tendríamos estados normales físicos negativos en amplitudes de dispersión. No puedo creer que fuera tan simple :-) ¡gracias! (deberías publicar eso como respuesta)
no se si es el unico motivo...
@ValterMoretti Supongo que esperaré en caso de que alguien tenga algo más que decir, pero para mí su comentario responde bastante a mi pregunta.
Un comentario adicional: la forma más sencilla de hacer una teoría de calibre invariante es tener campos que se transformen como una representación lineal no trivial del grupo de calibre. el grupo EM tu ( 1 ) no tiene representaciones reales unidimensionales no triviales, por lo que los escalares reales individuales no pueden acoplarse de esta manera. Hay formas más complicadas de acoplar un escalar real a un campo de calibre (bosón de Higgs, dilatón, axión, etc.) pero sus interacciones no son lo que cabría esperar ingenuamente.
Además, los campos de spin-1 sin simetría de calibre no se pueden volver a normalizar.

Respuestas (2)

El hecho de que la teoría no sea invariante de calibre implica que todos los grados de libertad de A m debe tener un significado físico: Esta no es la teoría de los fotones donde solo los grados transversales de libertad tienen sentido. De esta manera, debe abordar algún problema no trivial como la norma negativa asociada con los modos temporales. Esto podría evitarse agregando una masa a A m y dando vueltas 1 (en lugar de helicidad) a las partículas asociadas. Sin embargo, una vez más, este no es el campo EM.

ANEXO . En realidad, si le sumamos una masa a A m y asumimos que el campo describe partículas con espín 1 (evitando problemas con los modos temporales) la condición m A m = 0 debe agregarse solo para eliminar un grado de libertad (o incluso es automático si se usa la acción Proca según lo observado por AccidentalFourierTransform). Esto tiene la devastadora consecuencia de que la interacción lagrangiana se convierte en un término límite, es decir, se desvanece:

A m ϕ m ϕ d 4 X = 1 2 m ( A m ϕ 2 ) d 4 X
Creo que tenemos una teoría inútil desde cada punto de vista.

La teoría puede ser inútil en la física de alta energía, pero los términos límite pueden tener consecuencias físicas en la física de la materia condensada.
Tienes razón, lamentablemente no soy un experto en ese tema...
@ValterMoretti: Entonces, ¿cree que el sistema con campos reales descrito en mi respuesta es problemático? Y otra pregunta. ¿Las conclusiones de su nuevo trabajo arxiv.org/abs/1611.09029 son aplicables a este sistema? Estoy tratando de entender, no de criticar.
Sí, porque rompe la invariancia de calibre, que es fundamental en la teoría EM, ya que corresponde al hecho de que los fotones tienen solo dos grados de libertad internos. Con respecto a su última pregunta, mi artículo reciente con uno de mis estudiantes de doctorado, M. Oppio, se refiere más a la mecánica cuántica relativista que a la QFT, por lo que es difícil decir algo sin desarrollar más la teoría.
@ValterMoretti: No creo que rompa la invariancia del calibre más que cualquier otra elección de condición de calibre. En este caso se utiliza la condición de calibre unitario (el campo de materia es real).
Lo siento, te confundí a ti y al OP de la pregunta inicial. ¡Déjame pensar en tu respuesta!
Creo que, en principio, funciona. El único punto sutil que veo es que el procedimiento de fijación de su indicador depende del campo de materia que no se conoce, pero que solo se determina dinámicamente. En cambio, los procedimientos estándar para fijar el calibre del campo EM son independientes del campo de materia (piense en el calibre de Coulomb, Lorentz o Landau). Supongo que este hecho puede dar lugar a dificultades a la hora de cuantificar. Sin embargo, no sé, no soy un experto en estos temas... +1 sin embargo
@ValterMoretti: Gracias por tu opinión y el voto a favor. Puede haber un problema con la condición del indicador según el campo de la materia. Por otro lado, cualquier otra condición de calibre también fija el campo de materia, no solo el campo electromagnético. A menudo veo razonamientos que explican por qué la teoría cuántica requiere números complejos y trato de entender si el sistema de mi respuesta (o un sistema similar de Dirac-Maxwell) es compatible con dicho razonamiento y cómo. Es por eso que te hice la pregunta después de leer tu artículo reciente.

Consideremos la electrodinámica escalar (electrodinámica de Klein-Gordon-Maxwell) con el Lagrangiano:

1 4 F m v F m v + 1 2 ( ψ , m i mi A m ψ ) ( ψ , m + i mi A m ψ ) 1 2 metro 2 ψ ψ

y las ecuaciones de movimiento

( m + i mi A m ) ( m + i mi A m ) ψ + metro 2 ψ = 0 ,
A m A , v m v = j m ,
j m = i mi ( ψ ψ , m ψ , m ψ ) 2 mi 2 A m ψ ψ .

El complejo campo de materia cargada ψ puede hacerse real mediante una transformada de calibre (al menos localmente), y las ecuaciones de movimiento en el calibre relevante (calibrador unitario) para el cuatro potencial transformado del campo electromagnético B m y campo de materia real φ son como sigue ( mi .   S C h r o ¨ d i norte gramo mi r , norte a t tu r mi , 169:538, 1952):

φ ( mi 2 B m B m metro 2 ) φ = 0 ,
B m B , v m v = j m ,
j m = 2 mi 2 B m φ 2 .

Schrödinger hizo el siguiente comentario: "Que la función de onda ... pueda hacerse real mediante un cambio de calibre no es más que una perogrullada, aunque contradice la creencia generalizada de que los campos 'cargados' requieren una representación compleja".

Estas ecuaciones de movimiento se pueden obtener del Lagrangiano de trabajo (T. Takabayasi (1953), Progr. Theor. Phys., 9 ,187):

1 4 F m v F m v + 1 2 mi 2 B m B m ϕ 2 + 1 2 ( φ , m φ , m metro 2 φ 2 ) .
En realidad, coincide con el Lagrangiano anterior hasta la sustitución del campo escalar complejo por uno real. Cualquier solución de las ecuaciones de movimiento del primer Lagrangiano tiene una solución físicamente equivalente de las ecuaciones de movimiento del segundo Lagrangiano.

No consideré la cuantización aquí.

EDITAR (10/02/2018):

Me gustaría agregar que el sistema anterior de campo escalar real interactivo y campo electromagnético tiene algunas propiedades sorprendentes. A partir de las ecuaciones de movimiento se puede ver que el campo escalar real se puede eliminar algebraicamente, y resulta que las ecuaciones resultantes para el campo electromagnético describen su evolución independiente (mi artículo en European Physical Journal C http://link.springer. com/content/pdf/10.1140%2Fepjc%2Fs10052-013-2371-4.pdf y referencias allí).

Aparentemente, también es posible introducir un Lagrangiano invariante de Lorentz con derivadas superiores que no incluye el campo de la materia, pero es en gran medida equivalente al Lagrangiano de Takabayasi (mi artículo https://arxiv.org/abs/1006.2578 ; el significado de algunos casos especiales, por ejemplo, φ = 0 y B m B m = 0 (ver más abajo) no está claro (¿diferentes partículas?)). Con este fin, el Takabayasi Lagrangian se puede expresar en términos de Φ = φ 2 , más bien que φ , utilizando, por ejemplo, lo siguiente:

φ , m φ , m = 1 4 Φ , m Φ , m Φ ,
y entonces Φ puede ser reemplazada por la siguiente expresión obtenida de las ecuaciones de movimiento:
Φ = 1 2 mi 2 B m ( B m B , v m v ) B m B m .

Por lo tanto, un Lagrangiano que incluye solo un campo electromagnético describe prácticamente la misma física que la electrodinámica escalar.

Perdona por tardar tanto en comentar, he estado fuera un tiempo. Muchas gracias por tu respuesta :-) Si te entendí bien, estás diciendo que el Lagrangiano que escribiste empareja el B campo a un real ϕ , ¿Correcto? En ese caso, ¿debemos decir que ϕ esta cargada o descargada?
@AccidentalFourierTransform: Diría que en esta situación describe un campo cargado.